Bạn chưa đăng nhập. Vui lòng đăng nhập để hỏi bài

Những câu hỏi liên quan
Phạm Thị Duyên
Xem chi tiết
Đặng Ngọc Quỳnh
3 tháng 10 2020 lúc 21:34

Ta có: \(\left(x-y\right)^2\ge0\Leftrightarrow x^2+y^2\ge2xy\Leftrightarrow\left(x+y\right)^2\ge4xy\)

\(\Rightarrow4.2011a\left(2011a-2\right)\le\left(2011a+2011a-2\right)^2=4\left(2011a-1\right)^2\)

\(\Leftrightarrow2011a\left(2011a-2\right)\le\left(2011a-1\right)^2\)

\(\Leftrightarrow\frac{2011a\left(2011a-2\right)}{\left(2011a-1\right)^2}\le1\)

\(\Leftrightarrow\frac{1}{a}-\frac{2011a\left(2011a-2\right)}{\left(2011a-1\right)^2}\ge\frac{1}{a}-1\)\(\Leftrightarrow\frac{1}{a\left(2011a-1\right)^2}\ge\frac{1}{a}-1\)

Tương tự: \(\frac{1}{b\left(2011b-1\right)^2}\ge\frac{1}{b}-1;\frac{1}{c\left(2011c-1\right)^2}\ge\frac{1}{c}-1\)

\(\Leftrightarrow\frac{1}{a\left(2011a-1\right)^2}+\frac{1}{b\left(2011b-1\right)^2}+\frac{1}{c\left(2011c-1\right)^2}\ge\frac{1}{a}+\frac{1}{b}+\frac{1}{c}-3=2011-3=2008\)

Sai thì thôi nhá bẹn!

Khách vãng lai đã xóa
Neet
Xem chi tiết
Akai Haruma
2 tháng 3 2017 lúc 0:34

Bài 3)

BĐT cần chứng minh tương đương với:

\(\left ( \frac{a}{a+b} \right )^2+\left ( \frac{b}{b+c} \right )^2+\left ( \frac{c}{c+a} \right )^2\geq \frac{1}{2}\left ( 3-\frac{a}{a+b}-\frac{b}{b+c}-\frac{c}{c+a} \right )\)

Để cho gọn, đặt \((x,y,z)=\left (\frac{b}{a},\frac{c}{b},\frac{a}{c}\right)\) \(\Rightarrow xyz=1\).

BĐT được viết lại như sau:

\(A=2\left [ \frac{1}{(x+1)^2}+\frac{1}{(y+1)^2}+\frac{1}{(z+1)^2} \right ]+\frac{1}{x+1}+\frac{1}{y+1}+\frac{1}{z+1}\geq 3\) \((\star)\)

Ta nhớ đến hai bổ đề khá quen thuộc sau:

Bổ đề 1: Với \(a,b>0\) thì \(\frac{1}{(a+1)^2}+\frac{1}{(b+1)^2}\geq \frac{1}{ab+1}\)

Cách CM rất đơn giản, Cauchy - Schwarz:

\((a+1)^2\leq (a+b)(a+\frac{1}{b})\Rightarrow \frac{1}{(a+1)^2}\geq \frac{b}{(a+b)(ab+1)}\)

Tương tự với biểu thức còn lại và cộng vào thu được đpcm

Bổ đề 2: Với \(x,y>0,xy\geq 1\) thì \(\frac{1}{x^2+1}+\frac{1}{y^2+1}\geq \frac{2}{xy+1}\)

Cách CM: Quy đồng ta có đpcm.

Do tính hoán vị nên không mất tổng quát giả sử \(z=\min (x,y,z)\)

\(\Rightarrow xy\geq 1\). Áp dụng hai bổ đề trên:

\(A\geq 2\left [ \frac{1}{xy+1}+\frac{1}{(z+1)^2} \right ]+\frac{2}{\sqrt{xy}+1}+\frac{1}{z+1}=2\left [ \frac{z}{z+1}+\frac{1}{(z+1)^2} \right ]+\frac{2\sqrt{z}}{\sqrt{z}+1}+\frac{1}{z+1}\)

\(\Leftrightarrow A\geq \frac{2(z^2+z+1)}{(z+1)^2}+\frac{1}{z+1}+2-\frac{2}{\sqrt{z}+1}\geq 3\)

\(\Leftrightarrow 2\left [ \frac{z^2+z+1}{(z+1)^2}-\frac{3}{4} \right ]+\frac{1}{z+1}-\frac{1}{2}-\left ( \frac{2}{\sqrt{z}+1}-1 \right )\geq 0\)

\(\Leftrightarrow \frac{(z-1)^2}{2(z+1)^2}-\frac{z-1}{2(z+1)}+\frac{z-1}{(\sqrt{z}+1)^2}\geq 0\Leftrightarrow (z-1)\left [ \frac{1}{(\sqrt{z}+1)^2}-\frac{1}{(z+1)^2} \right ]\geq 0\)

\(\Leftrightarrow \frac{\sqrt{z}(\sqrt{z}-1)^2(\sqrt{z}+1)(z+\sqrt{z}+2)}{(\sqrt{z}+1)^2(z+1)^2}\geq 0\) ( luôn đúng với mọi \(z>0\) )

Do đó \((\star)\) được cm. Bài toán hoàn tất.

Dấu bằng xảy ra khi \(a=b=c\)

P/s: Nghỉ tuyển lâu rồi giờ mới gặp mấy bài BĐT phải động não. Khuya rồi nên xin phép làm bài 3 trước. Hai bài kia xin khiếu. Nếu làm đc chắc tối mai sẽ post.

Lightning Farron
2 tháng 3 2017 lúc 18:11

Bài 1:

Cho \(a=b=c=\dfrac{1}{\sqrt{3}}\). Khi đó \(M=\sqrt{3}-2\)

Ta sẽ chứng minh nó là giá trị nhỏ nhất

Thật vậy, đặt c là giá trị nhỏ nhất của a,b,c. Khi đó, ta cần chứng minh

\(\frac{a^2}{b}+\frac{b^2}{c}+\frac{c^2}{a}-\frac{2(a^2+b^2+c^2)}{\sqrt{ab+ac+bc}}\geq(\sqrt3-2)\sqrt{ab+ac+bc}\)

\(\Leftrightarrow\sqrt{ab+ac+bc}\left(\frac{a^2}{b}+\frac{b^2}{c}+\frac{c^2}{a}-\sqrt{3(ab+ac+bc)}\right)\geq2(a^2+b^2+c^2-ab-ac-bc)\)

\(\Leftrightarrow\frac{a^2}{b}+\frac{b^2}{a}-a-b+\frac{b^2}{c}+\frac{c^2}{a}-\frac{b^2}{a}-c+a+b+c-\sqrt{3(ab+ac+bc)}\geq\)

\(\geq2((a-b)^2+(c-a)(c-b))\)

\(\Leftrightarrow(a-b)^2\left(\frac{1}{a}+\frac{1}{b}-2\right)+(c-a)(c-b)\left(\frac{1}{a}+\frac{b}{ac}-2\right)+a+b+c-\sqrt{3(ab+ac+bc)}\geq0\)

Đúng bởi \(\frac{1}{a}+\frac{1}{b}-2>0;\frac{1}{a}+\frac{b}{ac}-2\geq\frac{1}{a}+\frac{1}{a}-2>0\)

\(a+b+c-\sqrt{3(ab+ac+bc)}=\frac{(a-b)^2+(c-a)(c-b)}{a+b+c+\sqrt{3(ab+ac+bc)}}\geq0\)

BĐT đã được c/m. Vậy \(M_{Min}=\sqrt{3}-2\Leftrightarrow a=b=c=\dfrac{1}{\sqrt{3}}\)

P/s: Nhìn qua thấy ngon mà làm mới thấy thật sự là "choáng"

Hung nguyen
2 tháng 3 2017 lúc 10:57

Câu 1/ Ta có

\(ab+bc+ca\le\frac{\left(a+b+c\right)^2}{3}\)

\(\Leftrightarrow1\le\frac{\left(a+b+c\right)^2}{3}\)

\(\Leftrightarrow\left(a+b+c\right)^2\ge3\)

\(\Leftrightarrow\sqrt{3}\le a+b+c< 3\)

Ta có: \(M=\frac{a^2\left(1-2b\right)}{b}+\frac{b^2\left(1-2c\right)}{c}+\frac{c^2\left(1-2a\right)}{a}\)

\(=\frac{a^2}{b}+\frac{b^2}{c}+\frac{c^2}{a}-2\left(a^2+b^2+c^2\right)\)

\(\ge\frac{\left(a+b+c\right)^2}{a+b+c}-2\left(a^2+b^2+c^2\right)-4\left(ab+bc+ca\right)+4\left(ab+bc+ca\right)\)

\(=a+b+c-2\left(a+b+c\right)^2+4\) (1)

Đặt \(a+b+c=x\left(\sqrt{3}\le x< 3\right)\)

Ta tìm GTNN của hàm số: \(y=-2x^2+x+4\)

\(\Rightarrow y'=-4x+1=0\)

\(\Rightarrow x=\frac{1}{4}=0,25\)

Thế x lần lược các giá trị \(\left\{\begin{matrix}x=0,25\\x=\sqrt{3}\end{matrix}\right.\)

\(\Rightarrow\left\{\begin{matrix}y=4,125\\y=-2+\sqrt{3}\end{matrix}\right.\)

\(\Rightarrow y_{min}=-2+\sqrt{3}\) đạt cực trị tại \(x=\sqrt{3}\) (2)

Từ (1) và (2) ta suy ra GTNN của M là \(-2+\sqrt{3}\) tại \(a=b=c=\frac{1}{\sqrt{3}}\)

Nguyễn Thiều Công Thành
Xem chi tiết
Lầy Văn Lội
13 tháng 8 2017 lúc 22:33

\(P=\frac{a^3}{\left(a+1\right)\left(b+1\right)}+\frac{b^3}{\left(b+1\right)\left(c+1\right)}+\frac{c^3}{\left(c+1\right)\left(a+1\right)}-1\)

Bá đạo sever là tao
13 tháng 8 2017 lúc 22:36

ôi trá hình :VVV

tth_new
14 tháng 8 2017 lúc 20:18

\(P=\frac{a^3}{\left(a+1\right).\left(b+1\right)}+\frac{b^3}{\left(b+1\right).\left(c+1\right)}+\frac{c^3}{\left(c+1\right).\left(a+1\right)}\)

Ko biết đúng hay không!

Mới lớp 6 , mà tôi nghĩ Lầy Văn Lội đúng đấy!

zZz Cool Kid zZz
Xem chi tiết
Hoàng Bình Minh
Xem chi tiết
Hoàng Đức Khải
Xem chi tiết
bach nhac lam
Xem chi tiết
Nguyễn Việt Lâm
11 tháng 2 2020 lúc 22:21

Mới nghĩ ra 3 câu:

a/ \(\frac{ab}{\sqrt{\left(1-c\right)^2\left(1+c\right)}}=\frac{ab}{\sqrt{\left(a+b\right)^2\left(1+c\right)}}\le\frac{ab}{2\sqrt{ab\left(1+c\right)}}=\frac{1}{2}\sqrt{\frac{ab}{1+c}}\)

\(\sum\sqrt{\frac{ab}{1+c}}\le\sqrt{2\sum\frac{ab}{1+c}}\)

\(\sum\frac{ab}{1+c}=\sum\frac{ab}{a+c+b+c}\le\frac{1}{4}\sum\left(\frac{ab}{a+c}+\frac{ab}{b+c}\right)=\frac{1}{4}\)

c/ \(ab+bc+ca=2abc\Rightarrow\frac{1}{a}+\frac{1}{b}+\frac{1}{c}=2\)

Đặt \(\left(x;y;z\right)=\left(\frac{1}{a};\frac{1}{b};\frac{1}{c}\right)\Rightarrow x+y+z=2\)

\(VT=\sum\frac{x^3}{\left(2-x\right)^2}\)

Ta có đánh giá: \(\frac{x^3}{\left(2-x\right)^2}\ge x-\frac{1}{2}\) \(\forall x\in\left(0;2\right)\)

\(\Leftrightarrow2x^3\ge\left(2x-1\right)\left(x^2-4x+4\right)\)

\(\Leftrightarrow9x^2-12x+4\ge0\Leftrightarrow\left(3x-2\right)^2\ge0\)

d/ Ta có đánh giá: \(\frac{x^4+y^4}{x^3+y^3}\ge\frac{x+y}{2}\)

\(\Leftrightarrow\left(x-y\right)^2\left(x^2+xy+y^2\right)\ge0\)

Khách vãng lai đã xóa
bach nhac lam
11 tháng 2 2020 lúc 21:42

Akai Haruma, Nguyễn Ngọc Lộc , @tth_new, @Băng Băng 2k6, @Trần Thanh Phương, @Nguyễn Việt Lâm

Mn giúp e vs ạ! Thanks!

Khách vãng lai đã xóa
Bùi Hữu Vinh
Xem chi tiết
Bùi Hữu Vinh
16 tháng 2 2021 lúc 23:14

giúp với 

Khách vãng lai đã xóa
pham trung thanh
Xem chi tiết
vũ tiền châu
27 tháng 12 2017 lúc 13:37

ta có A=\(\frac{a}{bc}+\frac{b}{ac}+\frac{c}{ab}+\frac{a^2}{2}+\frac{b^2}{2}+\frac{c^2}{2}=\frac{a^2+b^2+c^2}{abc}+\frac{a^2}{2}+\frac{b^2}{2}+\frac{c^2}{2}\)

mà \(a^2+b^2+c^2\ge ab+bc+ca\Rightarrow\frac{a^2+b^2+c^2}{abc}\ge\frac{ab+bc+ca}{abc}=\frac{1}{a}+\frac{1}{b}+\frac{1}{c}\)

\(\Rightarrow A\ge\frac{a^2}{2}+\frac{b^2}{2}+\frac{c^2}{2}+\frac{1}{a}+\frac{1}{b}+\frac{1}{c}=\frac{a^2}{2}+\frac{1}{2a}+\frac{1}{2a}+...\)

Áp dụng bđt co si ta có , \(\frac{a^2}{2}+\frac{1}{2a}+\frac{1}{2a}\ge\frac{1}{\sqrt{2}}\)

tương tự mấy cái kia rồi + vào thì A>=...